RegistrierenRegistrieren   LoginLogin   FAQFAQ    SuchenSuchen   
Dezentraler, elastischer Stoß
 
Neue Frage »
Antworten »
    Foren-Übersicht -> Mechanik
Autor Nachricht
willyengland



Anmeldungsdatum: 01.05.2016
Beiträge: 686

Beitrag willyengland Verfasst am: 12. Dez 2019 12:25    Titel: Dezentraler, elastischer Stoß Antworten mit Zitat

Hallo, habe mal wieder eine knifflige Aufgabe:

Ein Neonatom mit Masse m = 20 u, stößt vollständig elastisch mit einem anderen, ruhenden Atom der unbekannten Masse m_x zusammen.
Nach dem Aufprall bewegt sich das Neonatom in einem Winkel von θNe = 55,6° aus seiner ursprünglichen Richtung.
Das unbekannte Atom bewegt sich wie gezeigt in einem Winkel von θx = 50° weg.
Bestimmen Sie mit Hilfe der Energie-Impuls-Erhaltung: Um welches Edelgasatom handelt es sich beim Stoßpartner?


Den Impuls kann ich in die x und die y-Richtung aufteilen:





Außerdem habe ich noch die Energieerhaltung:



Damit habe ich 3 Gleichungen, aber 4 Unbekannte.
Wie rechne ich das jetzt konkret weiter?



1b.jpg
 Beschreibung:

Download
 Dateiname:  1b.jpg
 Dateigröße:  33.26 KB
 Heruntergeladen:  138 mal


_________________
Gruß Willy
Myon



Anmeldungsdatum: 04.12.2013
Beiträge: 5906

Beitrag Myon Verfasst am: 12. Dez 2019 15:41    Titel: Re: Dezentraler, elastischer Stoß Antworten mit Zitat

willyengland hat Folgendes geschrieben:
(...) Damit habe ich 3 Gleichungen, aber 4 Unbekannte.
Wie rechne ich das jetzt konkret weiter?

Hmm. Unbekannt sind doch m2, u1 und u2?
willyengland



Anmeldungsdatum: 01.05.2016
Beiträge: 686

Beitrag willyengland Verfasst am: 12. Dez 2019 15:45    Titel: Antworten mit Zitat

und v1
_________________
Gruß Willy
Myon



Anmeldungsdatum: 04.12.2013
Beiträge: 5906

Beitrag Myon Verfasst am: 12. Dez 2019 16:06    Titel: Antworten mit Zitat

Stimmt, ich hatte nach dem Lesen des Aufgabentexts wirklich gemeint, das sei gegeben. Dann sehe ich nicht, wie man die Aufgabe lösen kann, aber ich schau es mir heute abend noch einmal an.
willyengland



Anmeldungsdatum: 01.05.2016
Beiträge: 686

Beitrag willyengland Verfasst am: 12. Dez 2019 19:27    Titel: Antworten mit Zitat

Hallo Myon, danke!
Ich habe noch folgendes gefunden, es scheint mir auf mein Problem zu passen, aber ich bekomme es nicht auf die Reihe:

https://physics.stackexchange.com/questions/46089/elastic-collision-in-two-dimensions

https://physics.stackexchange.com/questions/159880/is-it-possible-to-determine-the-outcome-of-any-impact-knowing-only-the-ratio-of

Vielleicht kannst du damit was anfangen?

_________________
Gruß Willy
willyengland



Anmeldungsdatum: 01.05.2016
Beiträge: 686

Beitrag willyengland Verfasst am: 13. Dez 2019 07:58    Titel: Antworten mit Zitat

Ich hatte noch folgende Idee:
Letztlich ist es ja völlig egal, wie groß v1 ist. Die Winkel ändern sich dadurch ja nicht. Man könnte also einfach v1=1 annehmen.

Mal sehen ...

_________________
Gruß Willy
willyengland



Anmeldungsdatum: 01.05.2016
Beiträge: 686

Beitrag willyengland Verfasst am: 13. Dez 2019 09:36    Titel: Antworten mit Zitat

Habe es jetzt mal durchgerechnet.
Interessanterweise kürzt sich v1 raus!

Aber irgendwie ist da noch ein Fehler drin.
Der Faktor vor m1 ist negativ.
Wo ist der Fehler?



Berechnung.jpg
 Beschreibung:

Download
 Dateiname:  Berechnung.jpg
 Dateigröße:  808.04 KB
 Heruntergeladen:  133 mal


_________________
Gruß Willy
Neue Frage »
Antworten »
    Foren-Übersicht -> Mechanik